Number Theory 31

(Chọn đội tuyển KHTN 2009)
[Bài toán]: Tìm a,b,c,d\in \mathbb{N} đôi một phân biệt thỏa mãn: a^2-b^2=b^2-c^2=c^2-d^2

Lời Giải. Bài toán tương đương với việc tìm một cấp số cộng thực sự gồm 4 số chính phương. Ta chứng minh rằng không tồn tại một cấp số cộng như vậy. Giả sử ngược lại tồn tại 4 số chính phương A^2,B^2,C^2,D^2 lập thành một cấp số cộng tăng, tức là A^2-B^2=B^2-C^2. Trong các cấp số như thế, chọn cấp số có công sai nhỏ nhất. Ta có thể giả sử rằng các số chính phương này đôi một nguyên tố cùng nhau, và tính chẵn lẻ của các phương trình chứng tỏ rằng mỗi một số chính phương này phải lẻ. Như vậy tồn tại các số nguyên nguyên tố cùng nhau u,v sao cho A=u-v,C=u+v,u^2+v^2=B^2, và công sai của cấp số cộng bằng \displaystyle{\frac{C^2-A^2}{2}=2uv}

Ta cũng có D^2-B^2=4uv, và có thể viết thành \displaystyle{[\frac{D+B}{2}.\frac{D-B}{2}]}=uv. Hai thừa số ở vế trái nguyên tố cùng nhau, và u và v cũng thế. Như vậy tồn tại 4 số nguyên đôi một nguyên tố cùng nhau a,b,c,d (trong đó có đúng 1 số chẵn) sao cho u=ab,v=cd,D+B=2ac,D-B=2bd. Từ đây suy ra B=ac-bd, và như vậy ta có thể thế vào phương trình u^2+v^2=B^2 để được (ab)^2+(cd)^2=(ac-bd)^2. Phương trình này là đối xứng đối với 4 biến số nên ta có thể giả sử c là chẵn và a, b, d là lẻ. Từ phương trình bậc 2 này ta suy ra c là hàm hữu tỷ của căn bậc 2 của a^4-a^2d^2+d^4, từ đó suy ra tồn tại số nguyên lẻ m sao cho a^4-a^2d^2+d^2=m^2

Vì a và d là lẻ nên tồn tại các số nguyên nguyên tố cùng nhau x và y sao cho a^2=k(x+y) và d^2=k(x-y), trong đó k=\pm 1. Thay vào phương trình nói trên, ta được x^2+3y^2=m^2, từ đó rõ ràng là y phải là số chẵn và x lẻ. Đổi dấu x nếu cần, ta có thể giả sử m+x\vdots 3, ta có \displaystyle{3(\frac{y}{2})^2=\frac{(m+x)(m-x)}{4}}, từ đó suy ra \displaystyle{\frac{m+x}{2}} là ba lần số chính phương còn \displaystyle{\frac{m-x}{2}} là số chính phương. Như vậy ta có các số nguyên nguyên tố cùng nhau r, s (một số chẵn và một số lẻ) sao cho \displaystyle{\frac{3+x}{2}=3r^2,\frac{m-x}{2}=s^2,m=3r^2+s^2,x=3r^2-s^2, y=\pm 2rs}

Thay x và y vào các biểu thức của a^2,d^2 (và biến đổi nếu cần) ta được a^2=k(s+r)(s-3r) và d^2=k(s-r)(s+3r). Vì các thừa số ở vế phải là nguyên tố cùng nhau nên 4 đại lượng s-3r,s-r,s+r,s+3r phải có trị tuyệt đối chính phương, với công sai 2r. Các đại lượng này tất cả phải cùng dấu vì nếu ngược lại thì tổng của hai số chính phương lẻ bằng hiệu của hai số chính phương lẻ, tức là, 1+1\equiv 1-1\pmod 4, mâu thuẫn.

Vì thế, ta phải có \left | 3r \right |<s, và do m=3r^2+s^2 ta có 12r^2<m. Mặt khác, từ phương trình bậc 4 ta có m<a^2+d^2, như vậy ta có bất đẳng thức \left | 2r \right |<\left | max(a,d) \right |. Như vậy ta có 4 số chính phương lập thành một cấp số cộng với công sai \left | 2r \right |<\left | 2abcd \right |, và số cuối cùng chính là công sai của cấp số cộng ban đầu. Điều này mâu thuẫn với cách chọn bốn số chính phương ban đầu, phép chứng minh hoàn tất. \blacksquare
_________________________________

Nhận xét: Bài toán Fermat của học sinh, còn kia là bài toán Fermat của các GS,TS @@

Number Theory 27

(Middle European MO 2012)
[Bài toán]:Tìm x,y,z\in \mathbb{Z^+} thỏa mãn: \begin{cases}x^y+y^x=z^y\qquad (1)\\x^y+2012=y^{z+1}\qquad (2)\end{cases}
                                                              Lời giải
Từ (2)\Rightarrow x,y cùng dấu \Rightarrow z chẵn. Đặt z=2c với c nguyên dương

\blacktriangleright Xét x,y cùng chẵn, đặt x=2a,y=2b với a,b nguyên dương
Ta có: (2)\Leftrightarrow (2a)^y+2^2.503=(2b)^{z+1}\Leftrightarrow 2^{2b-2}a^y+503=2^{2c-1}b^{2c-1}
\Rightarrow 2^{2b-2}.a^y lẻ \Rightarrow a lẻ, 2^{2b-2}=1\Leftrightarrow b=1\Rightarrow y=2
Thay vào (1), ta có: x^2+2^x=z^2\Leftrightarrow 2^x=(z-x)(z+x)\Rightarrow \begin{cases}z-x=2^m\\z+x=2^n\end{cases} với m<n,m+n=x nguyên dương
• Với m\geq 2. Ta có: 2x=2^n-2^m\Leftrightarrow a=2^{m-2}(2^{n-m}-1)a lẻ
\Rightarrow m=2\Leftrightarrow z-x=4\Leftrightarrow z=4+x\Rightarrow 2^x=8x+16
Ta thấy với x\in \{1;...;5\} không thỏa mãn, x\geq 7\rightarrow 2^x>8x+16 (theo quy nạp) \Rightarrow x=6\rightarrow z=10. Thử lại thỏa mãn.
• Với m=1\Rightarrow z-x=2\Leftrightarrow z=2+x\Rightarrow 2^x=4x+4
Với x=1 không thỏa mãn, x\geq 2\Rightarrow x+1=2^{x-2}\Rightarrow x=2 (không thỏa mãn)

\blacktriangleright Xét x,y cùng lẻ. Trừ (1)-(2), ta có: y^x+y^{z+1}=z^y+2012\qquad (3)
• Với y=1\Rightarrow x=-2011 (vô lí)
• Với y\geq 3. Từ (1)\Rightarrow x^y<z^y\Rightarrow x\leq z+1
Ta có: (3)\Rightarrow y^x(1+y^{z+1-x})\equiv 0\pmod 4y lẻ \Rightarrow 4\nmid y^x, z+1-x chẵn, y lẻ
\Rightarrow 1+y^{z+1-x}\equiv 2\pmod 4\Rightarrow VT\equiv 2\pmod 4 (mâu thuẫn!)
Kết luận: Vậy \boxed{(x,y,z)=(6;2;10)} thỏa mãn ycđb \blacksquare

Number Theory 26

(Phát triển từ bài toán Taiwan 1999)
[Bài toán]: Cho b nguyên dương sao cho b+1=q là số nguyên tố. Gọi p là ước nguyên tố lớn nhất của b^y+1,q là ước nguyên tố nhỏ nhất của y. CMR: p\geq q+2.
                                                Lời giải
Phản chứng, nếu p\geq q, gọi \text{ord}_pb=h\qquad (h\in \mathbb{Z^+}) \qquad (1)
ta có: p|b^y+1\Leftrightarrow b^{2y}\equiv 1\pmod p \Longrightarrow h|2y\qquad (2)
Từ (1)\Rightarrow h|p-1\Rightarrow p-1\geq h\Leftrightarrow p\geq h+1;p\leq q\Rightarrow h+1\leq q\Rightarrow h<q\Rightarrow \gcd(y,h)=1\qquad (3)
Từ (2),(3)\Rightarrow h|2\Rightarrow h\in \{1;2\}
• Xét h=1\Rightarrow p|b-1. Ta có: p|b^y+1\Rightarrow p|2\Rightarrow p|2\Rightarrow p=2\Rightarrow b lẻ \Rightarrow q chẵn \Rightarrow q=2 (vô lí)
• Xét h=2\Rightarrow p|(b-1)(b+1)\Leftrightarrow p|q(q-2). Nếu \gcd(q,q-2)>1\Rightarrow q chẵn \Rightarrow q=2 (vô lí) \Longrightarrow \gcd(q,q-2)=1. Nếu p|q-2\Rightarrow p|b-1\Rightarrow q=2 (vô lí) \Longrightarrow p|q\Rightarrow q=pp là ước nguyên tố lớn nhất của b^y+1
\Rightarrow q là ước duy nhất của b^y+1. Đặt b^y+1=q^n với n\in \mathbb{Z^+}
Ta có phương trình: (q-1)^y+1=q^n\Rightarrow (q-1)^{2y}\equiv 1\pmod {q^n}. Đặt \text{ord}_{q^n}q-1=h với h\in \mathbb{Z^+}
\Rightarrow h|2y\text{ord}_q(q-1)=2\Rightarrow \text{ord}_{q^n}q-1=2q^{n-1}\Rightarrow 2q^{n-1}|2y \Leftrightarrow q^{n-1}|y\Rightarrow q|y. Đặt y=kq với k\in \mathbb{Z^+}
Dễ dàng thấy y lẻ, nếu y chẵn \Rightarrow q|2\Rightarrow q=2 (vô lí)
\Longrightarrow k lẻ. Ta thấy q^n=(q-1)^y+1=[(q-1)^q]^k+1=[(q-1)^q+1].A\vdots (q-1)^q+1q\in \mathbb{P}\Rightarrow \exists m\in \mathbb{Z^+}:(q-1)^q+1=q^m
q\geq 5\Rightarrow (q-1)^q+1\geq (q-1)^5+1\geq q^3\Rightarrow q^m\geq q^3\Leftrightarrow m\geq 3
Ta có: (q-1)^q+1=q^q-\text{C}_p^1q^{q-1}+...+q^2\equiv q^2\pmod {q^3}q^m\equiv 0\pmod {q^3}\forall m\geq 3\Rightarrow VT\neq NP (mâu thuẫn). Vậy giả thiết phản chứng sai \Rightarrow p>qp,q nguyên tố \Rightarrow p\geq q+2
Kết luận: Vậy ta có Q.E.D \blacksquare

Number Theory 23

(VMO 2004) 
[Bài toán]: Tìm x,y,z\in \mathbb{Z^+} sao cho (x+y)(1+xy)=2^z\qquad (1)
                                                                Lời Giải
Không mất tính tổng quát, giả sử y\geq x.
Từ (1)\Rightarrow (I)\begin{cases}x+y=2^m\\1+xy=2^{z-m}\end{cases} với m\in \mathbb{Z^+}
Ta thấy x+y-1-xy=(x-1)(1-y)\leq 0x,y\in \mathbb{Z^+}\Rightarrow 2^m\leq 2^{z-m}\Leftrightarrow z\geq 2m
\blacktriangleright Xét x=1\Rightarrow \begin{cases}y=2^m-1\\y+1=2^{z-m}\end{cases}\Rightarrow 2^m=2^{z-m}\Leftrightarrow z=2m
Do đó, phương trình có nghiệm (x,y,z)=(1;2^m-1;2m) với m\in \mathbb{Z^+}
\blacktriangleright Xét x\geq 2. Từ (I)\Rightarrow x,y lẻ \Rightarrow x,y\geq 3
Xét biểu thức: y(x-1)(x+1)=yx^2-y=yx^2+x-x-y=x(xy+1)-(x+y)
=2^{z-m}x-2^m=2^m(2^{z-m}x-1)
\Rightarrow 2^m\mid y(x-1)(x+1)\gcd(y,2^m)=1\Rightarrow 2^m\mid (x+1)(x-1)
\Rightarrow \exists x+1,x-1: 2^{m-1}\mid x+1,2^{m-1}\mid x-1
Nếu 2^{m-1}\mid x-1\Rightarrow 2^{m-1}+1\leq x;x+y=2^m\qquad (3)
\Leftrightarrow y=2^m-x\leq 2^m-2^{m-1}-1=2^{m-1}-1\qquad (2)
Từ (2),(3)\Rightarrow y<x (trái với giả thiết)
\Rightarrow 2^{m-1}\mid x+1\Rightarrow x\geq 2^{m-1}-1;x+y=2^m\qquad (4)
\Leftrightarrow y=2^m-x\leq 2^m-2^{m-1}+1=2^{m-1}+1\qquad (5)
Từ (4),(5)\Rightarrow 2^{m-1}-1\leq x\leq y\leq 2^{m-1}+1
x,y lẻ; x\neq yx=y\Rightarrow x+y\neq 2^m (dễ CM)
\Rightarrow \begin{cases}x=2^{m-1}-1\\y=2^{m-1}+1\end{cases}\Rightarrow 1+(2^{m-1}-1)(2^{m-1}+1)=2^{z-m}
\Leftrightarrow 2^{2m-2}=2^{z-m}\Leftrightarrow 2m-2=z-m\Leftrightarrow z=3m-2
x\geq 3\Rightarrow 2^{m-1}-1\geq 3\Leftrightarrow 2^{m-1}\geq 2^2\Leftrightarrow m\geq 3
Kết luận: Phương trình có nghiệm nguyên dương là:
(x,y,z)\in \{(1;2^{m-1};2m);(2^{m-1};1;2m)\} với m\in \mathbb{Z^+}
(x,y,z)\in \{(2^{m-1}-1;2^{m+1}+1;3m-2);(2^{m+1}+1;2^{m-1}-1;3m-2)\} với m\geq 3,m \in \mathbb{Z^+} \blacksquare

Number Theory 21

(IMO 1997)
[Bài toán]: Tìm nghiệm nguyên dương của phương trình: a^{b^2}=b^a
                                                              Lời giải 
Đặt \gcd(a,b)=d\Rightarrow a=dm,b=dn(\gcd(m,n)=1)
\Rightarrow [(dm)^{dn^2}]^d=[(dn)^m]^d\Rightarrow (dm)^{dn^2}=(dn)^m\Rightarrow d^{dn^2}.m^{dn^2}=d^m.n^m(1)
\blacktriangleright TH1: dn^2=m\Rightarrow m^{dn^2}=n^m\gcd(m,n)=1\Rightarrow m=n=1dn^2=m\Rightarrow d=1\Rightarrow (a,b)=(1;1)
\blacktriangleright TH2: dn^2>m\Rightarrow d^{dn^2-m}.m^{dn^2}=n^m
Thấy m^{dn^2}\mid n^m\gcd(m,n)=1\Rightarrow m=1
\Rightarrow d^{dn^2-1}=n^1=n
• Xét d=1\Rightarrow n=1 \Rightarrow (a,b)=(1;1)
• Xét d\geq 2\Rightarrow n\geq 2. Dùng quy nạp, ta sẽ CM: 2^{2n^2-1}>n\qquad (2)
Giả sử (2) \text{True}\forall n=k, ta sẽ CM \forall n=k+1,(3) vẫn đúng.
Thật vậy, ta có: 2^{2k^2-1}>k\Rightarrow 2^{2k^2-1}\geq k+1
Ta có: 2^{2(k+1)^2-1}=2^{2k^2+4k+1}=2^{2k^2-1}.2^{4k+2}\geq (k+1).2^{4k+2}\geq k+1
Do đó (2) được CM \Rightarrow Trường hợp này không xảy ra.
\blacktriangleright TH3: Xét dn^2<m\Rightarrow (2)\Leftrightarrow m^{dn^2}=d^{m-dn^2}.n^m \Rightarrow n^m\mid m^{dn^2}\gcd(m,n)=1\Rightarrow n=1
\Rightarrow (2)\Leftrightarrow m^d=d^{m-d}. Theo bài Number Theory 8
\Rightarrow (m;d)\in \{(1;1);(9;3);(8;2)\}\Rightarrow (a,b)\in \{(1;1);(27;3);(16;2)\}
Kết luận: Nghiệm của phương trình là (a,b)\in \{(1;1);(27;3);(16;2)\} \blacksquare

Number Theory 20

(Junior Balkan MO 2000)
[Bài toán] Tìm n\in \mathbb{Z^+} sao cho n^2+3^n là số chính phương.
                                                           Lời Giải
Đặt n^2+3^n=a^2(1)(a\in \mathbb{Z^+})
\Leftrightarrow (a-n)(a+n)=3^n \Rightarrow\begin{cases}a-n=3^m\\ a+n=3^{n-m}\end{cases} (m\in \mathbb{Z^+})
Ta có: m<c-m\Leftrightarrow 2m<c\Leftrightarrow 2m+1\leq c
\blacktriangleright Xét n=2m+1 \Rightarrow \begin{cases}a-n=3^m\\a+n=3^{m+1}\end{cases}
\Rightarrow a+n=3(a-n)
\Leftrightarrow a=2n\Rightarrow n^2+3^n=4n^2
\Leftrightarrow 3^n=3n^2\Leftrightarrow 3^{n-1}=n^2 \Rightarrow n lẻ.
Đặt n=2k+1(k\in \mathbb{N}) \Rightarrow (3^k)^2=(2k+1)^2\Leftrightarrow 3^k=2k+1
Áp dụng BĐT Bernulli: 1+2k\leq (1+2)^k=3^k
\Rightarrow k\in \{0;1\}\Rightarrow n\in \{1;3\} (Thử lại thỏa mãn)
\blacktriangleright Xét n>2m+1\Rightarrow n\geq 2m+2
Ta có: 2n=3^{n-m}-3^m=9.3^{n-m-2}-3^m3^m\leq 3^{n-m-2}\Leftrightarrow n\geq 2m+2(\text{True})
Áp dụng bất đẳng thức Bernulli:
\Rightarrow 2n\geq 8.3^{n-m-2}=8.(1+2)^{n-m-2}\geq 8[1+2(n-m-2)]=16n-16m-24
\Leftrightarrow 7n\leq 8m+127n\geq 14m+14\Rightarrow 6m+2\leq 0 (vô lí)
Vậy \boxed{n\in \{1;3\}} thỏa mãn \blacksquare

Number Theory 15

(Olympic Chuyên KHTN 2014)
Tìm tất cả bộ 3 số (x,n,p) với x,n\in \mathbb{Z^+},p\in \mathbb{P} thỏa mãn:
x^3+2x=3(p^n-1)
                                                             Lời Giải
Ta có: x^3+2x=3(p^n-1) \Leftrightarrow (x+1)(x^2-x+3)=3p^n
Đặt \gcd(x+1;x^2-x+3)=d(d\in \mathbb{N^*})
\Rightarrow d\mid x+1;d\mid x^2-x+3=(x+1)(x-2)+5 d\mid 5 d\in \begin{Bmatrix} 1;5 \end{Bmatrix}
(*) Xét d=1
+, Xét p=3\Rightarrow (x+1)(x^2-x+3)=3^{n+1}=1.3^{n+1}x\in \mathbb{Z^+}\Rightarrow x+1>1, x^2-x+3>1 \Rightarrow Trường hợp này không xảy ra
+, Xét p\neq 3
\begin{cases} x+1=3 \\ x^2-x+3=p^n \end{cases} \Leftrightarrow \begin{cases}n=1 \\ x=2 \\ p=5 \end{cases}
\begin{cases} x+1=p^n \\ x^2-x+3=3 \end{cases} \Leftrightarrow \begin{cases}x=1 \\ p=2 \\ n=1 \end{cases}
(*) Xét d=5\Rightarrow 3p^n\vdots 5 \Rightarrow p=5, với n\geq 2
Ta có: (x+1)(x^2-x+3)=3.5^n=3.5.5^{n-1} nên xét các trường hợp sau:
\begin{cases} x+1=5 \\ x^2-x+3=3.5^{n-1}\end{cases} \Leftrightarrow \begin{cases} x=4\\ n=2\\ p=5\end{cases}
• \begin{cases} x+1=3.5^{n-1} \\ x^2-x+2=5\end{cases} \Leftrightarrow \begin{cases} x=2\\ n=1\\ p=5\end{cases}
• \begin{cases} x+1=3.5 \\ x^2-x+3=5^{n-1}\end{cases} \Leftrightarrow \begin{cases} x=14\\ 5^{n-1}=185\end{cases} (vô lí)
• \begin{cases} x+1=5^{n-1} \\ x^2-x+3=15\end{cases} \Leftrightarrow \begin{cases} x=4\\ n=2\\ p=5\end{cases}
Vậy phương trình có nghiệm \boxed{(x,n,p)=\{ (4;2;5); (1;1;2); (2;1;5)\}} \blacksquare

Number Theory 12

(VMO 1995)
Tìm tất cả các số nguyên a,b,n>1 thỏa mãn điều kiện: (a^3+b^3)^n=4(ab)^{1995} \qquad (1)
                                                        Lời Giải
Từ (1)\Rightarrow a^3+b^3 chẵn \Rightarrow a,b cùng dấu.
Đặt a=2^px_p; b=2^qy_p (với p,q\geq 0;x_p,y_p lẻ)
(*) Xét p=q=0\Rightarrow a,b lẻ
+, Nếu n>2 \Rightarrow 8\mid (a^3+b^3)^n8\nmid 4(ab)^{1995}
\Rightarrow Trường hợp này không xảy ra.
+, Nếu n=2\Rightarrow (a^3+b^3)^2=4(ab)^{1995}
\Leftrightarrow a^6+b^6=2(ab)^3[2(ab)^{1992}-1] 2(ab)^{1992}-1\mid a^6+b^6a^6+b^6>0
\Rightarrow a^6+b^6\geq 2(ab)^{1992}-1 \Leftrightarrow \displaystyle{\frac{1}{a^6}+\frac{1}{b^6}+\frac{1}{a^6b^6}\geq 2(ab)^{1986}} (vô lí vì a,b>1)
(*) Xét p,q>0\Rightarrow a,b chẵn
Ta có: (1)\Leftrightarrow [(2^px_p)^3+(2^qy_p)^3]^n=4(2^{p+q}x_py_p)^{1995}
\Leftrightarrow [(2^{p-1}x_p)^3+(x^{q-1}y_p)^3]^n=2^{1995(p+q)+2+3n}.(x_py_p)^{1995}
\Leftrightarrow .......
\Leftrightarrow [(2^{p-k}x_p)^3+(2^{q-k}y_p)^3]^n=2^{1995(p+q)+2+3n.2^{k-1}}.(x_py_p)^{1995} (với k\in \mathbb{Z^+},k\leq p,q)
Xét p\neq q, vì vai trò như nhau, giả sử p>q. Nếu q=k\Rightarrow VT lẻ mà VP chẵn.
\Rightarrow Trường hợp này không xảy ra p=q
\Rightarrow [(2^{p-1})^3(x^3_p+y^3_p)]^n=2^{3990p+2+3n}.(x_py_p)^{1995}
\Leftrightarrow (x^3_p+y^3_p)^n=2^{3990p+2+6n-3np}.(x_py_p)^{1995}
x_p,y_p lẻ \Rightarrow (x^3_p+y^3_p)^n chẵn
Đặt (x^3_p+y^3_p)^n=2^nl (l lẻ, l\in \mathbb{Z^+})
Ta có: 2^nl=2^{3990p+2+6n-3np}.(x_py_p)^{1995}
\Leftrightarrow l=2^{3990p+2+5n-3np}.(x_py_p)^{1995}(x_py_p)^1995 lẻ
\Rightarrow 2^{3990p+2+5n-3np}=1 \Leftrightarrow 3990p+2+5n-3np=0
\Leftrightarrow (3p-5)(n-1330)=6652
p,n\in \mathbb{Z^+} \Rightarrow 3p-5;n-1330\in \mathbb{Z} \qquad (2)
Từ (1),n>1\Rightarrow 4(ab)^{1995}=(a^3+b^3)^n\geq 2^n(ab)^{\frac{3}{2}} \geq 4ab^{\frac{3n}{2}}
\displaystyle{\Rightarrow \frac{3n}{2}\leq 1995} \Leftrightarrow n\leq 1330
• Xét =1\Rightarrow n-1330=-3326 \Leftrightarrow n<0 (vô lí)
• Xét p\geq 2\Rightarrow 3p-5>0 \qquad (3)
Từ  (2),(3),6652=2^2.1663\Rightarrow Ta có các trường hợp sau:
-Nếu 3p-5=1,n-1330=6652\Rightarrow p=2,n=7982 (loại)
-Nếu 3p-5=2,n-1330=3326\Rightarrow \displaystyle{p=\frac{7}{3}},n=4656 (loại)
-Nếu 3p-5=4,n-1330=1663\Rightarrow p=3,n=2993 (loại)
-Nếu 3p-5=1663,n-1330=4\Rightarrow p=556,n=1334 (loại)
-Nếu 3p-5=3326,n-1330=2\Rightarrow \displaystyle{p=\frac{3331}{3}},n=1332 (loại)
-Nếu 3p-5=6652,n-1330=1\Rightarrow p=2219,n=1331 (loại)
Kết luận: Vậy phương trình không có nghiệm nguyên (a,b,n)>1  \blacksquare

Number Theory 13

(Balkan MO 2005)
[Bài toán]: Tìm p nguyên tố sao cho p^2-p+1 là lập phương một số tự nhiên.
                                                          Lời Giải
Đặt p^2-p+1=a^3(a\in \mathbb{N}) \qquad (1)
-Với a=0 không thỏa mãn
(1) \Leftrightarrow p(p-1)=(a-1)(a^2+a+1)\qquad (2)
\Rightarrow p\mid (a-1)(a^2+a+1) \Rightarrow 2 trường hợp:
\blacktriangleright Nếu p\mid a-1. Đặt a-1=pk(k\in \mathbb{N})
Ta có: (1)\Leftrightarrow p^2-p+1=(pk+1)^3=p^3k^3+3p^2k^2+3pk+1
\Leftrightarrow p-1=p^2k^3+3pk^2+3k
+, Nếu k\geq 2 \Rightarrow p-1\geq 8p^2+12p+6\Leftrightarrow 8p^2+11p+7\leq 0 (vô lí)
+, Nếu k=1 \Rightarrow p-1=p^2+3p+3 \Leftrightarrow p^2+2p+4=0 (vô lí)
+, Nếu k=0 \Rightarrow p=1 (vô lí)
Vậy trường hợp này không xảy ra.
\blacktriangleright Nếu p\mid a^2+a+1 \qquad (3)
Từ (2) \Rightarrow a-1\mid p(p-1)\gcd(p;p-1)=1 nên ta có 2 trường hợp:
+, Xét a-1\mid p \Rightarrow p\geq a-1 \Rightarrow p^2-p+1\leq (p+1)^3
\Leftrightarrow p(p^2+2p+4)\leq 0 (vô lí)
+, Xét a-1\mid p-1. Đặt p-1=q(a-1)(q\in \mathbb{Z^+}) \Leftrightarrow p=q(a-1)+1\qquad (4)
Từ (3),(4) \Rightarrow \displaystyle{\frac{a^2+a+1}{aq-q+1}}\in \mathbb{Z^+}
\Rightarrow \displaystyle{\frac{a^2+aq+q}{aq-q+1}=\frac{a(aq-q+1)+2(aq-q+1)+3q-a-2}{aq-q+1}}
=a+2+\displaystyle{\frac{3q-a-2}{aq-q+1}}\in \mathbb{Z^+}
\Rightarrow aq-q+1\mid 3q-a-2
\Rightarrow \left | 3q-a-2 \right |\geq aq-q+1
• Nếu 3q-a-2\geq aq-q+1 \Leftrightarrow q(4-a)\geq 3+a \Leftrightarrow (q+1)(a-4)\leq -7
Thay a=1;2;3 đều không thỏa mãn \Rightarrow a\geq 4 \Rightarrow (q+1)(a-4)\geq 0 (vì q\in \mathbb{Z^+}). Ta có điều mâu thuẫn nên trường hợp này không xảy ra.
• Nếu 2+a-3q\geq aq-q+1 \Leftrightarrow a+1\geq q(a+2) (vô lí vì a+1<a+2,q\in \mathbb{Z^+})
• Nếu 2+a-3q=0 \Leftrightarrow 2+a=3q
Ta có: (4)\Leftrightarrow 3p=3q(a-1)+3=(a-1)(2+a)+3=a^2+a+1
\Rightarrow (2)\Leftrightarrow p(p-1)=3p(a-1) \Leftrightarrow p=3a-2
\Rightarrow 3(3a-2)=a^2+a+1\Leftrightarrow (a-1)(a-7)=0\Leftrightarrow a=1;7
\Rightarrow a=7\Rightarrow p=19 (Thay vào (1))
Kết luận: Vậy \boxed {p=19} thỏa mãn \blacksquare

 

Number Theory 4

Giải phương trình nghiệm nguyên: a^2+b^2+c^2=\frac{5}{3}(ab+bc+ca)
                                                                     Lời giải
Ta có: a^2+b^2+c^2=\frac{5}{3}(ab+bc+ca)
\Leftrightarrow 3(a^2+b^2+c^2)=5(ab+bc+ca)
\Leftrightarrow 3a^2-5a(b+c)+(3b^2+3c^2-5bc)=0 (1)
Coi phương trình trên bậc hai ẩn a.
Ta có: \Delta_{a}=[5(b+c)]^2-4.3.(3b^2+3c^2-5bc)
=25b^2+50bc+25c^2-36b^2-36c^2+60bc=-11b^2-11c^2+110bc
=-11(b^2+c^2-10bc)=-11(b^2-10bc+25c^2-24c^2)=11[-(b-5c)^2+24c^2]
Để phương trình (1) có nghiệm nguyên thì \Delta_{a} là số chính phương
Đặt $latex \Delta_{a}=k^2(k\in \mathbb{N})$
Ta có: 11[24c^2-(b-5c)^2]=k^2\Rightarrow k^2\vdots 11
\Rightarrow k\vdots 11. Đặt k=11q(q\in \mathbb{N})
Ta có: 24c^2-(b-5c)^2=11q^2\Leftrightarrow 24c^2=11q^2+(b-5c)^2 (*)
Nhận xét các số chính phương chia 11 chỉ có dư là 0;1;3;4;5;9 (1)
\Rightarrow (b-5c)^2\equiv 0;1;3;4;5;9\pmod {11}
\Rightarrow 24c^2\equiv 0;1;3;4;5;9\pmod {11}
\Rightarrow c^2\equiv 0;2;6;8;10;7\pmod {11} (2)
Từ (1) và (2) \Rightarrow c^2\equiv 0\pmod {11}\Rightarrow c\vdots 11
Xét phương trình (*),  đặt b-5c=x
Ta có: (*)\Leftrightarrow x^2+11k^2=24c^2
c\vdots 11\Rightarrow c=11c_{1}(c_{1}\in \mathbb{Z})
Từ (*)\Rightarrow x^2\vdots 11\Rightarrow x\vdots 11\rightarrow x=11x_{1}(x_{1}\in \mathbb{Z})
Ta có: (*)\Leftrightarrow 121x_{1}^2+11k^2=24.11^2.c_{1}^2
\Leftrightarrow 11x_{1}^2+k^2=24.11c^2_{1}
\Rightarrow k^2\vdots 11 \Rightarrow k\vdots 11\rightarrow k=11k^2_{1}
Ta có: (*)\Leftrightarrow x^2_{1}+11k_{1}^2=24c^2_{1}
Quá trình cứ lặp đi lặp lại như vậy, theo nguyên lí lùi vô hạn, ta có bộ nghiệm tầm thường
(x,k,c)=(x_1,k_1,c_1) =...=(0;0;0)
\Rightarrow (a,b,c)=(0;0;0)
Vậy phương trình có nghiệm nguyên là (a,b,c)=(0;0;0)